6
$\begingroup$

Motivation. Yesterday I was sitting idly in the train, contemplating the train network. I noticed that a lot of lines (not all) intersected, and some pairs of lines intersected in quite a few stations. I wondered whether in general it was possible to minimize the intersections in a fictitious train network in which all lines intersected.

Formalization. Let $X\neq \emptyset$ be a set. We say ${\cal C}\subseteq {\cal P}(X)$ is a cover if $\bigcup {\cal C} = X$, and it is intersecting if whenever $A,B\in{\cal C}$ then $A\cap B \neq \emptyset$.

If ${\cal C}, {\cal D} \subseteq {\cal P}(X)$, we say ${\cal C}$ refines ${\cal D}$, or ${\cal C} \preceq {\cal D}$, if for all $C \in {\cal C}$ there is $D\in {\cal D}$ with $C\subseteq D$. Note that $\preceq$ is reflexive and transitive, but not anti-symmetric in general.

We say that an intersecting cover ${\cal C}_0$ is refinement-minimal if whenever ${\cal C}$ is an intersecting cover with ${\cal C} \preceq {\cal C}_0$, then ${\cal C}_0\preceq {\cal C}$.

Question. If $X$ is a non-empty set and ${\cal C} \subseteq {\cal P}(X)$ is an intersecting cover of $X$, is there necessarily an intersecting cover ${\cal C}_0\preceq {\cal C}$ such that ${\cal C}_0$ is refinement-minimal?

$\endgroup$
6
  • 1
    $\begingroup$ I would try to approach this with Zorns lemma. I would like to consider the set of intersecting covers of $X$ as a poset, but as you stated, refining is not antisymmetric. Thus we can pass to equivalence classes of intersecting covers where $C\sim D$ iff $C\le D$ and $D\le C$. Then I would think about the assumption on chains in Zorns lemma. $\endgroup$ Commented Apr 17, 2024 at 8:18
  • 1
    $\begingroup$ Is there an obvious answer in the special case where $X=\omega$ and $\mathcal C$ is an intersecting cover whose elements are finite sets? In this case I don't think Zorn's lemma or the axiom of choice would come up. But maybe this case is trivial and I'm just not clever enough to see it. $\endgroup$ Commented Apr 18, 2024 at 3:14
  • 1
    $\begingroup$ As an example of an intersecting cover of $\omega$ by finite sets which has no linear refinement, let $\mathcal C$ consist of the sets $\{0,2,4\}$, $\{1,3,4\}$, and $\{0,1,n\}$ where $n\ge5$. Note that $\mathcal C$ is a refinement-minimal cover and is not linear. $\endgroup$ Commented Apr 19, 2024 at 0:53
  • 1
    $\begingroup$ If $\mathcal C$ is an intersecting cover of $X$ which is linear and has the property that, for each $A\in\mathcal C$, there is a unique element $a\in A$ which is covered by no other element of $\mathcal C$, then $\mathcal C$ is a minimal cover. Is that what you meant? $\endgroup$ Commented Apr 19, 2024 at 2:58
  • 1
    $\begingroup$ Ah, yes, thanks @bof, that's what I meant, but I wrote it in a wrong way. $\endgroup$ Commented Apr 19, 2024 at 7:44

3 Answers 3

5
$\begingroup$

Let us recall that $\mathfrak u$ is the smallest cardinality of a base of a free ultrafilter on $\omega$. It is known (and easy to see) that $\omega_1\le\mathfrak u\le\mathfrak c$.

Example. There exists an intersecting cover $\mathcal C$ of a set $X$ of cardinality $|X|=\mathfrak u$ such that every intersecting cover $\mathcal C_0$ of $X$ with $\mathcal C_0\preceq \mathcal C$ is not refinement-minimal.

Proof. Take any free ultrafilter $\mathcal U$ on $\omega$ with base $\mathcal B$ of cardinality $|\mathcal B|=\mathfrak u$. Consider the set $X=\omega\cup\mathcal B$ and observe that $\mathcal C:=\{\{B\}\cup B:B\in\mathcal B\}$ is an intersecting cover of $X$.

Now take any intersecting cover $\mathcal C_0$ of $X$ with $\mathcal C_0\preceq \mathcal C$.

Claim 1. For every set $C\in\mathcal C_0$, the set $C\cap \omega$ belongs to the ultrafilter $\mathcal U$.

Proof. In the opposite case, the set $\omega\setminus C$ belongs to the ultrafilter $\mathcal U$. Since $\mathcal C_0\preceq\mathcal C$, there exists a set $B\in\mathcal B$ such that $C\subseteq \{B\}\cup B$. Since $\omega\setminus C\in\mathcal U$ and $B\in\mathcal B\subseteq\mathcal U$, there exists a set $B'\in\mathcal B\setminus\{B\}$ such that $B'\subseteq B\setminus C$. Since $\mathcal C_0$ is a cover of $X=\mathcal B\cup\omega$, there exists an element $C'\in\mathcal C_0$ such that $B'\in C'$. Since $\mathcal C_0\preceq \mathcal C$, there exists a set $B''\in\mathcal B$ such that $B'\in C'\subseteq \{B''\}\cup B''$, which implies that $B''=B'$ and hence $C'\subseteq \{B'\}\cup B'$ and $$C\cap C'\subseteq C\cap (\{B\}\cup B)\cap (\{B'\}\cup B')=C\cap B\cap B'\subseteq C\cap (B\setminus C)=\emptyset,$$ which contradicts the intersection property of the family $\mathcal C_0$. $\quad\square$

Now take any set $B\in\mathcal B$ and consider the subfamily $\mathcal C_B:=\{C\in\mathcal C_0:B\in C\}$.

Claim 2. For every $C\in\mathcal C_B$ we have $B\in C\subseteq \{B\}\cup B$.

Proof. Since $\mathcal C_0\preceq \mathcal B$, there exists a set $B'\in\mathcal B$ such that $B\in C\subseteq\{B'\}\cup B'$, which implies $B=B'$ and hence $B\in C\subseteq \{B\}\cup B$. $\quad\square$

Choose any set $C'\in\mathcal C_B$ and consider the set $U':=\omega\cap\bigcup\mathcal C_B$, which belongs to the ultrafilter $\mathcal U$, by Claim 1. Choose a base $\mathcal D$ of the ultrafilter $\mathcal U':=\{U'\cap U:U\in\mathcal U\}$ such that $U'=\bigcup\mathcal D$, $\mathcal D\preceq\mathcal C_B$, but $C'\setminus\{B\}\not\subseteq D$ for every $D\in\mathcal D$. Consider the family $$\mathcal C'_0:=(\mathcal C_0\setminus\mathcal C_B)\cup\{\{B\}\cup D:D\in\mathcal D\}.$$ It is easy to see that $\mathcal C_0'$ is an intersecting cover of $X$ such that $\mathcal C_0'\preceq\mathcal C_0$. Assuming that $\mathcal C_0\preceq\mathcal C_0'$, we can find a set $D\in\mathcal D$ such that $C'\subseteq\{D\}\cup D$ and hence $C'\setminus\{B\}\subseteq D$, which contradicts the choice of the base $\mathcal D$. This contradiction shows that the intersecting cover $\mathcal C_0$ is not refinement-minimal. $\quad\square$

The above Example motivates the following problem (of exchange the uncountable cardinal $\mathfrak u$ by the cardinal $\omega$):

Problem. Is there an intersecting cover without refinement-minimal refinements on a countable set?

$\endgroup$
3
$\begingroup$

Disclaimer: This is not an answer, and just too long for a comment.

Here is an example why the chain condition does not hold (which does not mean that there are no refinement-minimal covers, just that Zorns lemma cant be used to construct them. It might be interesting to check by hand what happens for this cover.

Let $X=\{a,b\}\cup \mathbb{N}$ with cover $\{A,B\}$ with $A=\{a\}\cup \mathbb{N}$,$B=\{b\}\cup \mathbb{N}$. Let $A_i= \{a\}\cup \{i,...\}$. Then $\{A_{i+1},B\}$ is a refinement of $\{A_i,B\}$ and so we have a chain.

Suppose we had a cover $\mathcal{C}$ with $\mathcal{C}\le \{A_i,B\}$ for all $i$. Let $C\in \mathcal{C}$ be a set containing $a$ and thus $C$ cannot be a subset of $B$. By definition of the covers, we have $C\subset A_i$ for all $i$ and thus $C=\{a\}$. Hence $C$ and $B$ do not intersect and thus $\mathcal{C}$ cannot be intersecting.

However the given cover has a refinement-minimal intersecting cover. For example the collection of all subsets containing $0$ and not containing both $\{a,b\}$ should be one (if I am not mistaken).

I think this makes the poset quite interesting. It has minimal elements, but the chain condition from Zorns lemma does not hold.

$\endgroup$
0
2
$\begingroup$

This is not even a partial answer, just an oversized comment. I consider only the special case where the set $X$ is countable and the intersecting cover $\mathcal C$ consists of finite sets. I am not able to settle even this special case, but I would like to record some equivalences which might be useful in settling it.

Let $\mathbf F$ be the set of all intersecting covers of $\mathbb N$ consisting of finite sets.

Lemma. For any $\mathcal C\in\mathbf F$ there exists $\mathcal C_1\in\mathbf F$ such that $\mathcal C_1\preceq\mathcal C$ and $\mathcal C_1$ is an antichain (no element of $\mathcal C_1$ is a subset of another).

Proof. Let $\mathcal M$ be the set of all minimal elements of $\mathcal C$ and let $\mathcal C_0=\{A\cup\{x\}:A\in\mathcal M,\ \{A\cup\{x\}\}\preceq C\}$. Then $\mathcal M\subseteq\mathcal C_0\preceq\mathcal C$ and $\mathcal C_0\in\mathbf F$. Let $\mathcal C_1$ be the set of all maximal elements of $\mathcal C_0$. Plainly $\mathcal C_1$ is an antichain and an intersecting family, and $\mathcal C_1\preceq\mathcal C$; I have to show that $\mathcal C_1$ covers $\mathbb N$. In fact I will show that there is no infinite chain in $\mathcal C_0$, whence every element of $\mathcal C_0$ is contained in a maximal element.

Assume for a contradiction that $\mathcal C_0$ contains an infinite chain $$A_1\cup\{x_1\}\subsetneqq A_2\cup\{x_2\}\subsetneqq A_3\cup\{x_3\}\subsetneqq\cdots$$ where $A_n\in\mathcal M$. Now $x_n\notin A_n$ for $n\ge2$, since $A_n\cup\{x_n\}\notin\mathcal M$ for $n\ge2$. Moreover, since $A_1\subseteq A_n\cup\{x_n\}$, while $A_1\not\subseteq A_n$ for $n\ge3$, we have $x_n\in A_1$ for $n\ge3$. Since $A_1$ is finite, by the pigeonhole principle we have $x_m=x_n=x$ for some $m,n$ with $3\le m\lt n$. But now, since $A_m\cup\{x\}\subsetneqq A_n\cup\{x\}$ and $x\notin A_m$, we have $A_m\subsetneqq A_n$, which is absurd.

Theorem. The following statements are equivalent:
(1) For any $\mathcal C\in\mathbf F$ there exists $\mathcal C'\in\mathbf F$, with $\mathcal C'\preceq\mathcal C$, such that $$\mathcal C'\succeq\mathcal B\in\mathbf F\implies\mathcal C'\preceq\mathcal B;$$ i.e., $\mathcal C'$ is refinement-minimal.
(2) For any $\mathcal C\in\mathbf F$ there exists $\mathcal C'\in\mathbf F$, with $\mathcal C'\preceq\mathcal C$, such that $$\mathcal C'\succeq\mathcal B\in\mathbf F\implies\mathcal C'\subseteq\mathcal B.$$
(3) For any $\mathcal C\in\mathbf F$ and any $x\in\mathbb N$ there exist $\mathcal C'$ and $A$, with $x\in A\in\mathcal C'\in\mathbf F$ and $\mathcal C'\preceq\mathcal C$, such that $$\mathcal C'\succeq\mathcal B\in\mathbf F\implies A\in\mathcal B.$$

Proof. Plainly (2)$\implies$(1) & (3); I will show that (1)$\implies$(2) and (3)$\implies$(2).

Proof of (1)$\implies$(2). Suppose $\mathcal C\in\mathbf F$. By (1) there exists $\mathcal C_0\in\mathbf F$ with $\mathcal C_0\preceq\mathcal C$ such that $\mathcal C_0$ is refinement-minimal. By the lemma there exists $\mathcal C'\in\mathbf F$ such that $\mathcal C'\preceq\mathcal C_0$ and $\mathcal C'$ is an antichain. Now, if $\mathcal C'\succeq\mathcal B\in\mathbf F$, then $\mathcal C'\preceq\mathcal B$; since $\mathcal C'$ is an antichain, it follows that $\mathcal C'\subseteq\mathcal B$.

Proof of (3)$\implies$(2). Let $\mathcal C_0=\mathcal C$. Using (3) we can construct $\mathcal C_n\in\mathbf F$ and $A_n\in\mathcal C_n$ for $n\in\mathbb N$ so that $\mathcal C_n\preceq\mathcal C_{n-1}$ and $n\in A_n$, and so that $\mathcal C_n\succeq\mathcal B\in\mathbf F\implies A_n\in\mathcal B$. It follows that $A_n\in\mathcal A_m$ for all $m\ge n$. Let $\mathcal C'=\{A_n:n\in\mathbb N\}$. Then $\mathcal C'\in\mathbf F$, and $\mathcal C'\preceq\mathcal C_n$ for all $n$, and $\mathcal C'\succeq\mathcal B\in\mathbf F\implies\mathcal C'\subseteq\mathcal B$.

Remark. I wanted to include the following statement but I couldn't prove that it's equivalent to the others:

(4) For any $\mathcal C\in\mathbf F$ and any $x\in\mathbb N$ there exist $\mathcal C'$ and $A$, with $x\in A\in\mathcal C'\in\mathbf F$ and $\mathcal C'\preceq\mathcal C$, such that $$\mathcal C'\succeq\mathcal B\in\mathbf F\implies\{A\}\preceq\mathcal B.$$

$\endgroup$
1
  • $\begingroup$ Thanks a lot @bof for this effort! I find statement (4) particularly intriguing. (Obviously one should extend the definition of $\preceq$ to all non-empty subsets of ${\cal P}(X)$, not just covers, because $\{A\}$ intersecting, but usually not a cover.) Also, I will have to think which of the implications between (1),(2),(3) hold in the general setting, not just ${\bf F}$. (Of course some implications are trivial.) $\endgroup$ Commented Apr 23, 2024 at 7:00

You must log in to answer this question.

Start asking to get answers

Find the answer to your question by asking.

Ask question

Explore related questions

See similar questions with these tags.